Difference between revisions of "2023 AMC 12A Problems/Problem 24"

(Solution 2)
(Solution 3 (Cheese, but this time it actually works))
Line 25: Line 25:
 
Seeing that all the answers are different modulus 5, and that 10 is divisible by 5, we cheese this problem.  
 
Seeing that all the answers are different modulus 5, and that 10 is divisible by 5, we cheese this problem.  
  
Let <math>A_1, A_2, \cdots, A_n</math> be one sequence satisfying the constraints of the problem. Let <math>b_1, b_2, \cdots, b_n, b_{n+1}</math> be the sequence of nonnegative integers such that <math>A_k</math> has <math>b_k</math> elements for all <math>1\leq{}k\leq{}n</math>, and <math>b_{n+1}=10</math>. Note that we can generate the number of valid sequences of <math>A</math> by first generating all sequences of <math>b</math> such that <math>b_i\leq{}b_{i+1}</math> for all <math>1\leq{}i<n</math>, then choosing the elements from <math>A_{k+1}</math> that we keep in <math>A_k</math>, given the sequence of <math>b</math> as the restraint for the number of elements. For each sequence <math>b_1, b_2, \cdots{}, b_{n+1}</math>, there are <math>\prod_{i=2}^{n+1}\binom{b_i}{b_{i-1}}</math> corresponding sequences for <math>A</math>. Now, consider two cases - either all terms in <math>b</math> are either 0 or 10, or there is at least one term in <math>b</math> that is neither 0 nor 10. In the second case, consider the last term in <math>b</math> that is not 10, say <math>b_m</math>. However, that implies <math>b_{m+1}=10</math>, and so the number of corresponding sequences of <math>A</math> is <math>\binom{10}{b_m}\cdot{}</math> something, which is a multiple of <math>5</math> because <math>\binom{10}{k}\equiv0</math> (mod 5) for all <math>k\neq{}0</math> and <math>k\neq{}10</math>. Therefore, we only need to consider sequences of <math>b</math> where each term is <math>0</math> or <math>10</math>. For each <math>1\leq{}n\leq{}10</math>, there are <math>n+1</math> sequences of <math>b</math> (since we have <math>n+1</math> places for <math>b</math> to turn from 0 to 10). Since each sequence of <math>b</math> here corresponds to exactly one sequence of <math>A</math> (since <math>\binom{10}{10}=\binom{10}{0}=\binom{0}{0}=1</math>), the answer is just sum of <math>n+1</math> for <math>2\leq{}n\leq{}10</math>. This is <math>2+3+\cdots+11=65\equiv0</math> (mod 5), so the answer is <math>\boxed{\textbf{(C) } 5}</math>.
+
Let <math>A_1, A_2, \cdots, A_n</math> be one sequence satisfying the constraints of the problem. Let <math>b_1, b_2, \cdots, b_n, b_{n+1}</math> be the sequence of nonnegative integers such that <math>A_k</math> has <math>b_k</math> elements for all <math>1\leq{}k\leq{}n</math>, and <math>b_{n+1}=10</math>. Note that we can generate the number of valid sequences of <math>A</math> by first generating all sequences of <math>b</math> such that <math>b_i\leq{}b_{i+1}</math> for all <math>1\leq{}i<n</math>, then choosing the elements from <math>A_{k+1}</math> that we keep in <math>A_k</math>, given the sequence of <math>b</math> as the restraint for the number of elements. For each sequence <math>b_1, b_2, \cdots{}, b_{n+1}</math>, there are <math>\prod_{i=2}^{n+1}\binom{b_i}{b_{i-1}}</math> corresponding sequences for <math>A</math>. Now, consider two cases - either all terms in <math>b</math> are either 0, 5, or 10, or there is at least one term in <math>b</math> that is neither 0, 5 nor 10. In the second case, consider the last term in <math>b</math> that is not 10 or 5, say <math>b_m</math>. However, that implies <math>b_{m+1}=10</math>, and so the number of corresponding sequences of <math>A</math> is <math>\binom{10}{b_m}\cdot{}</math> something or <math>\binom{5}{b_m}\cdot</math> something, which is always a multiple of <math>5</math>. Therefore, we only need to consider sequences of <math>b</math> where each term is <math>0</math> <math>5</math> or <math>10</math>. If all terms in <math>b</math> are 0 or 10, then for each <math>1\leq{}n\leq{}10</math> there are <math>n+1</math> sequences of <math>b</math> (since there are <math>n+1</math> places to turn from <math>0</math> to <math>10</math>), for a total of <math>2+3+\cdots+11=65\equiv0</math> (mod 5). If there exists at least one term <math>b_k=5</math>, then we use stars and bars to count the number of sequences of <math>b</math>, and each sequence of <math>b</math> corresponds to <math>\binom{10}{5}</math> sequences of <math>A</math>. For each <math>n</math>, we must have at least one term of <math>5</math>. After that, there are <math>n-1</math> stars and <math>2</math> bars (separating <math>0</math> to <math>5</math> and <math>5</math> to <math>10</math>), so that is <math>\binom{n+1}{2}</math> sequences of <math>b</math>. So the sum is <math>\binom{2}{2}+\binom{3}{2}+\cdots+\binom{11}{2}=\binom{12}{3}\equiv0</math> (mod 5). Therefore, the answer is 0 mod 5, and it must be <math>\boxed{\textbf{(C) } 5}</math>
  
 
==Solution==
 
==Solution==

Revision as of 23:17, 11 November 2023

Problem

Let $K$ be the number of sequences $A_1$, $A_2$, $\dots$, $A_n$ such that $n$ is a positive integer less than or equal to $10$, each $A_i$ is a subset of $\{1, 2, 3, \dots, 10\}$, and $A_{i-1}$ is a subset of $A_i$ for each $i$ between $2$ and $n$, inclusive. For example, $\{\}$, $\{5, 7\}$, $\{2, 5, 7\}$, $\{2, 5, 7\}$, $\{2, 5, 6, 7, 9\}$ is one such sequence, with $n = 5$.What is the remainder when $K$ is divided by $10$?

$\textbf{(A) } 1 \qquad \textbf{(B) } 3 \qquad \textbf{(C) } 5 \qquad \textbf{(D) } 7 \qquad \textbf{(E) } 9$

Solution 1

Consider any sequence with $n$ terms. Every 10 number has such choices: never appear, appear the first time in the first spot, appear the first time in the second spot… and appear the first time in the $n$th spot, which means every number has $(n+1)$ choices to show up in the sequence. Consequently, for each sequence with length $n$, there are $(n+1)^{10}$ possible ways.

Thus, the desired value is $\sum_{i=1}^{10}(i+1)^{10}\equiv \boxed{\textbf{(C) } 5}\pmod{10}$

~bluesoul

Solution 2

Let $f(x,\ell)$ be the number of sequences $A_1$, $A_2$, $\dots$, $A_\ell$ such that each $A_i$ is a subset of $\{1, 2,\dots,x\}$, and $A_i$ is a subset of $A_{i+1}$ for $i=1$, $2\dots$, $\ell-1$. Then $f(x,1)=2^x$ and $f(0,\ell)=1$.

If $\ell\ge2$ and $x\ge1$, we need to get a recursive formula for $f(x,\ell)$: If $|A_1|=i$, then $A_1$ has $\text C_x^i$ possibilities, and the subsequence $\{A_i\}_{2\le i\le\ell}$ has $f(x-i,\ell-1)$ possibilities. Hence \[f(x,\ell)=\sum_{i=0}^x\text C_x^if(x-i,\ell-1).\] By applying this formula and only considering modulo $10$, we get $f(1,2)=3$, $f(1,3)=4$, $f(1,4)=5$, $f(1,5)=6$, $f(1,6)=7$, $f(1,7)=8$, $f(1,8)=9$, $f(1,9)=0$, $f(1,10)=1$, $f(2,2)=9$, $f(2,3)=6$, $f(2,4)=5$, $f(2,5)=6$, $f(2,6)=9$, $f(2,7)=4$, $f(2,8)=1$, $f(2,9)=0$, $f(2,10)=1$, $f(3,2)=7$, $f(3,3)=4$, $f(3,4)=5$, $f(3,5)=6$, $f(3,6)=3$, $f(3,7)=2$, $f(3,8)=9$, $f(3,9)=0$, $f(3,10)=1$, $f(4,2)=1$, $f(4,3)=6$, $f(4,4)=5$, $f(4,5)=6$, $f(4,6)=1$, $f(4,7)=6$, $f(4,8)=1$, $f(4,9)=0$, $f(4,10)=1$, $f(5,2)=3$, $f(5,3)=4$, $f(5,4)=5$, $f(5,5)=6$, $f(5,6)=7$, $f(5,7)=8$, $f(5,8)=9$, $f(5,9)=0$, $f(5,10)=1$, $f(6,2)=9$, $f(6,3)=6$, $f(6,4)=5$, $f(6,5)=6$, $f(6,6)=9$, $f(6,7)=4$, $f(6,8)=1$, $f(6,9)=0$, $f(6,10)=1$, $f(7,2)=7$, $f(7,3)=4$, $f(7,4)=5$, $f(7,5)=6$, $f(7,6)=3$, $f(7,7)=2$, $f(7,8)=9$, $f(7,9)=0$, $f(7,10)=1$, $f(8,2)=1$, $f(8,3)=6$, $f(8,4)=5$, $f(8,5)=6$, $f(8,6)=1$, $f(8,7)=6$, $f(8,8)=1$, $f(8,9)=0$, $f(8,10)=1$, $f(9,2)=3$, $f(9,3)=4$, $f(9,4)=5$, $f(9,5)=6$, $f(9,6)=7$, $f(9,7)=8$, $f(9,8)=9$, $f(9,9)=0$, $f(9,10)=1$, $f(10,2)=9$, $f(10,3)=6$, $f(10,4)=5$, $f(10,5)=6$, $f(10,6)=9$, $f(10,7)=4$, $f(10,8)=1$, $f(10,9)=0$, $f(10,10)=1$.

Lastly, we get $K\equiv\textstyle\sum\limits_{i=1}^{10}f(10,i)\equiv\boxed{\textbf{(C) } 5}\pmod{10}$. ~Quantum-Phantom

Solution 3 (Cheese, but this time it actually works)

Seeing that all the answers are different modulus 5, and that 10 is divisible by 5, we cheese this problem.

Let $A_1, A_2, \cdots, A_n$ be one sequence satisfying the constraints of the problem. Let $b_1, b_2, \cdots, b_n, b_{n+1}$ be the sequence of nonnegative integers such that $A_k$ has $b_k$ elements for all $1\leq{}k\leq{}n$, and $b_{n+1}=10$. Note that we can generate the number of valid sequences of $A$ by first generating all sequences of $b$ such that $b_i\leq{}b_{i+1}$ for all $1\leq{}i<n$, then choosing the elements from $A_{k+1}$ that we keep in $A_k$, given the sequence of $b$ as the restraint for the number of elements. For each sequence $b_1, b_2, \cdots{}, b_{n+1}$, there are $\prod_{i=2}^{n+1}\binom{b_i}{b_{i-1}}$ corresponding sequences for $A$. Now, consider two cases - either all terms in $b$ are either 0, 5, or 10, or there is at least one term in $b$ that is neither 0, 5 nor 10. In the second case, consider the last term in $b$ that is not 10 or 5, say $b_m$. However, that implies $b_{m+1}=10$, and so the number of corresponding sequences of $A$ is $\binom{10}{b_m}\cdot{}$ something or $\binom{5}{b_m}\cdot$ something, which is always a multiple of $5$. Therefore, we only need to consider sequences of $b$ where each term is $0$ $5$ or $10$. If all terms in $b$ are 0 or 10, then for each $1\leq{}n\leq{}10$ there are $n+1$ sequences of $b$ (since there are $n+1$ places to turn from $0$ to $10$), for a total of $2+3+\cdots+11=65\equiv0$ (mod 5). If there exists at least one term $b_k=5$, then we use stars and bars to count the number of sequences of $b$, and each sequence of $b$ corresponds to $\binom{10}{5}$ sequences of $A$. For each $n$, we must have at least one term of $5$. After that, there are $n-1$ stars and $2$ bars (separating $0$ to $5$ and $5$ to $10$), so that is $\binom{n+1}{2}$ sequences of $b$. So the sum is $\binom{2}{2}+\binom{3}{2}+\cdots+\binom{11}{2}=\binom{12}{3}\equiv0$ (mod 5). Therefore, the answer is 0 mod 5, and it must be $\boxed{\textbf{(C) } 5}$

Solution

We observe that in each sequence, if element $e \in A_i$, then $e \in A_j$ for all $j \geq i$. Therefore, to determine a sequence with a fixed length $n$, we only need to determine the first set $A_i$ that each element in $\left\{ 1, 2, \cdots , 10 \right\}$ is inserted into, or an element is never inserted into any subset.

We have \begin{align*} K & = \sum_{n = 1}^{10} \left( n + 1 \right)^{10} \\ & = \sum_{m = 2}^{11} m^{10} . \end{align*}

Modulo 10, we have \begin{align*} K & \equiv \sum_{m = 2}^{11} m^2 \\ & \equiv \sum_{m = 1}^{11} m^2 - 1^2 \\ & \equiv \frac{11 \cdot \left( 11 + 1 \right) \left( 2 \cdot 11 + 1 \right)}{6} - 1\\ & \equiv 505 \\ & \equiv \boxed{\textbf{(C) 5}}  . \end{align*}

~Steven Chen (Professor Chen Education Palace, www.professorchenedu.com)

Video Solution 1 by OmegaLearn

https://youtu.be/0LLQW0XCKsQ

Video Solution

https://youtu.be/FpagLq1uzBI

~Steven Chen (Professor Chen Education Palace, www.professorchenedu.com)


See also

2023 AMC 12A (ProblemsAnswer KeyResources)
Preceded by
Problem 23
Followed by
Problem 25
1 2 3 4 5 6 7 8 9 10 11 12 13 14 15 16 17 18 19 20 21 22 23 24 25
All AMC 12 Problems and Solutions

The problems on this page are copyrighted by the Mathematical Association of America's American Mathematics Competitions. AMC logo.png